Where is the intermediary conclusion?
"Many local businesses depend on our region's natural beauty, and the heavy industrial activity o...
AndrewArabie on June 1 at 04:39PM
  • June 2019 LSAT
  • SEC3
  • Q20
2
Replies
Premise in question
So would the sentence in questions be another example of the necessary Premise for the argument ?
Nativeguy on April 27 at 06:29PM
  • June 2019 LSAT
  • SEC3
  • Q8
1
Reply
How is this statement considered evidence ?
This seems to happens to me quiet often, I notice the LSAT incorporates the word evidence into a ...
Nativeguy on April 27 at 06:07PM
  • June 2019 LSAT
  • SEC3
  • Q20
2
Replies
Why not A?
Because the premises lead us to assume 30 people cannot be hired, shouldn't we be able to conclud...
alyj628 on January 15, 2023
  • June 2019 LSAT
  • SEC3
  • Q9
1
Reply
"must have known"
Is the phrase "he must've known" not speculative? I read that not as a premise/fact but just sort...
AndrewArabie on November 14, 2022
  • June 2019 LSAT
  • SEC3
  • Q18
1
Reply
Why not D?
Can someone go over why D is wrong?
Jasmin1 on October 3, 2022
  • June 2019 LSAT
  • SEC3
  • Q17
1
Reply
Wrong Answers
Can someone go over how the other answers were wrong? B & C through me off a bit
Jasmin1 on October 3, 2022
  • June 2019 LSAT
  • SEC3
  • Q14
1
Reply
Can you explain this one?
I am having a hard time understanding why the conclusion for this one is answer choice B, instead...
DruHarris on October 3, 2022
  • June 2019 LSAT
  • SEC3
  • Q5
1
Reply
D
When I negate D, it also breaks the argument - please walk me through how you choose E over D wit...
nagelm on January 13, 2022
  • June 2019 LSAT
  • SEC3
  • Q15
1
Reply
Why C
Why is C correct?
sprozes on August 9, 2021
  • June 2019 LSAT
  • SEC3
  • Q16
4
Replies
Why is A correct?
Why is A correct? Can we look at the method for the strengthen question? Thanks
Isaac-Jeantete on May 24, 2021
  • June 2019 LSAT
  • SEC3
  • Q14
4
Replies
Help
Could you please explain the correct answer?
yckim2180 on May 14, 2021
  • June 2019 LSAT
  • SEC3
  • Q19
1
Reply
Could someone please explain this?
Could someone please explain this? Thanks
jingjingxiao11111@gmail.com on September 26, 2020
  • June 2019 LSAT
  • SEC3
  • Q19
3
Replies
Why not B?
I narrowed to A and B
ruchitaj on August 7, 2020
  • June 2019 LSAT
  • SEC3
  • Q8
1
Reply
Why not A?
I narrowed to D and A
ruchitaj on August 7, 2020
  • June 2019 LSAT
  • SEC3
  • Q25
1
Reply
June 2019 LSAT lr 18
Can someone please explain this question? I sometimes confuse strengthen with sufficient premise ...
kens on August 6, 2020
  • June 2019 LSAT
  • SEC3
  • Q18
2
Replies
Help Please :)
I'm a little confused about why A is the correct answer. I thought that "and our sun has an unusu...
tkj97 on July 14, 2020
  • June 2019 LSAT
  • SEC3
  • Q8
1
Reply
Explanation
I don't get this one. Please explain. Thanks.
avif on July 7, 2020
  • June 2019 LSAT
  • SEC3
  • Q25
3
Replies
June 2019 SEC 3 Q17
I have the same question as the earlier post--please help! Thanks
kens on June 17, 2020
  • June 2019 LSAT
  • SEC3
  • Q17
1
Reply
Why is A correct?
Hi I didn't pick A because I thought not necessarily people get their lycopene from fruits and ve...
jingjingxiao11111@gmail.com on June 17, 2020
  • June 2019 LSAT
  • SEC3
  • Q17
1
Reply